Recherche sur le blog!

Concours Physique Concours Commun M Physique II 1994 (Corrigé)

Deuxième épreuve Physique II. Mines 94.
Première partie.
1.1. chaque composante Ei ou Bi vérifie l’équation différentielle
Δ Ei + (ω/c)2 Ei = 0 (1)
Les composantes tangentielles du champ électrique E sont nulles sur les parois.
Les composantes normales du champ B sont nulles sur les parois.
Par exemple
paroi, X1 = o : E2 = E3 = o B1 = o
paroi, X1 = a1 : E2 = E3 = o B1 = o
1.2. Le champ proposé vérifie les conditions aux limites. L’application de (1) à l’une des composantes de E donne :
ω2/c2 = π2 Σ (mi/ai)2

1.3. div.3. = o (absence de charge dans la cavité) entraîne :
Σ (mi Ei°/ai) = o (2) soit K.E = 0
Le champ électrique est perpendiculaire au vecteur K.
Pour une pulsation donnée le vecteur champ est déterminé par deux vecteurs de base perpendiculaire à K.
Remarquons aussi que la relation 2 impose une relation entre les Ei ; donc le champ dépend de deux composantes seulement.
1.4. Les deux miroirs sont parallèles et supposés infinis.
L’invariance par translation suivant x2 et x3 entraîne que le champ ne dépend que de x1.
Si on ne s’intéresse qu’aux ondes qui se propagent suivant x1, alors
E1 = o (champ transverse)
E2 = E°2 sin (m1 π x/a1) exp jωt
E3 = E°3 sin (m1 π x/a1) exp jωt
avec m1 π/a1 = ω/c
L’analogie mécanique est le mouvement d’une corde sans raideur liée à ses deux extrémités.
1.5.
dU ω = (ε°/u) [BB*.c2+ E.E* ]. dτ
Il suffit d’intégrer sur le volume.
1.6.
∫∫∫ B.B* dτ = - (1/jω) ∫∫∫ rot E.B*
= (1/jω [∫∫∫ div (E Λ B*) dτ + ∫∫∫ (E.rot.B*) dτ
rot B* = µo εο ((d*E*/dt) = - jω/c2 E*
d’où ∫∫∫ BB* dτ = - 1/jω [∫∫ (E Λ B*). n ds - jω/c2 ∫∫∫. E.E*
or, sur la surface E = o donc
∫∫∫ BB* dτ = 1/c2 ∫∫∫ ( E.E*) dτ
d’où l’expression :
d U = ε°/2 ( E.E*) dτ = u dτ
u = ε°/2 [ (E1 E1*+ E2E2* + E3E3*)
en prenant une valeur moyenne dans l’espace.
<u > = ε°/16 [ (E°1)2 + (E°2)2 + (E°3)2 ]

1.7.
Dans le domaine visible 400 nm < L < 800 nm, calculons l’ordre de grandeur du volume pour lequel on ait M grand.
V = (λ4/8πdλ) M = 1,3.10-20M
Per exemple si M = 100, V = 10-18 m3
Donc une cavité de côté a = 10-6m, il y a 100 modes possibles.
Deuxième partie.
2.1.1. On peut admettre qu’un miroir sphérique est équivalent à une lentille mince dont l’espace du rayon émergent est caractérisé par un milieu d’indice n = -1.
On peut également considérer que le foyer image et le foyer objet se confondent lorsqu’on replie le milieu émergent d’une lentille sur le milieu incident.
La distance focale de chaque lentille est donc R1/2 et R2/2.
Donc N aller-retours dans la cavité est équivalent à la propagation du rayon lumineux dans N motifs {L1 - L2 } , les deux lentilles sont distantes de L.
2.2.1.
α1 (p) = tan α1 (p) = (y2p - y1p) /L.
α2 (p) = (yp+11 - yp2) /L.
2.2.2. soient deux points conjugués A et A’ (A’ est virtuel) sur l’axe.
1/O1A’ - 1/O1A = 1/f’1.
avec α2(p-1) = - y1p/O1A α1p = - yp1/O1A’
d’où α1p - α2p-1 = - y1p/f’1 α2p - α1p = - y1p/f’2.
2.2.3. A partir de 2.2.1. et 2.2.2. , on obtient :
y2p-1 + y2p+1 + (2-u1 u2) y2p = o
soit f’1 f’2 (y2p-1 + y2p+1) + [ 2L (f’1+f’2) - 2f’1f’2 - L2] y2p = o
2.3.1. α1 (p+1) = (y2p+1 - y1p+1 )/L α1 (P) = (y2p - y1p)/L.
d’où y2p+1 - y1p+1 = - y2p + y1p
y2p+1 = - y2p
d’où α1 (p)- α2 (p)=( y1p - y2p + y1p+1 - y2p )= - (2 y2p/L) = - (y1p/f’2)
d’où f’2 = L/2
De même, on montre que les 2 conditions entraînent y2p+1 = - y2p
y2p - y1p - y1p + y2p-1 = - 2 y1p /L = - y1p /f’1
d’où f’1 = L/2.
La construction est classique. Remarquons que
F’1(p) = F2 (p) ; F’2 (p) = F1 (p+1)
F’1 (p+1) = F2 (p+1)

2.3.2. Les distances focales sont égales ; les foyers sont confondus.
Donc, C2 confondu avec S1 et C1 avec S2 .
Cavité confocale : S1 C1 = - S2C2 = L.
2.3.3. On trouve facilement que A et A’ sont confondus après un aller et retour.
2.3.4. En étudiant la progression selon la méthode du 2.3.1., on obtient
. après un aller-retour A’B’ = - AB
. après deux aller-retours A’B ’’ = AB.
On peut aussi le montrer par les relations classiques.
Relation des sinus d’Abbe AB. α = - A’B’ . α’.
AB -- M1 -- A1 B1 -- M2 -- A’B’.
Relation de Newton : FA. FA1 = f2 FA’.FA1 = f2
donc FA=FA’ AB = -A’B’
2.4.1. On remplace dans l’équation
y2p-1 + y2p+1 + (2-u1u2) y2p = o
il vient y2p [4 cos2 (φ/2) - u1u2] = o
donc cos2 φ/2 = u1u2/4
d’autre part o < cos2 (φ/2) < 1.
d’où o < 4 + L2/f’1f’2 - 2L [1/f’1 + 1/f’2] L4.
D’autre part, y2p doit être telle que son module soit inférieure à D/2 exprimant que les rayons restent confinés.
donc si A et A’ sont des réels, alors (A) + (A’) < D/2
2.4.2. Pour une cavité focale y2p+1 = y2p
Dans ce cas, rien n’est possible que si φ = (2k+1) π
D’autre part, si f’1 = f’2 = L/2 u1 = u2 = o et cos2 φ/2 = o
donc φ = π.
Troisième partie.
3.1.1. Pertes par diffusion et absorption si le milieu n’est pas le vide parfait.
Les miroirs diffractent ; une partie de la lumière est alors envoyée à
à l’extérieur de la cavité.
3.1.2. Un faisceau plan monochromatique λ arrivant sur un miroir plan de côté D
diffracte. La tache centrale est localisée dans un cône d’angle θ =λ/D.
Pour que tous les rayons lumineux situés dans ce cône atteigne l’autre
miroir, il faut que θ < D/L.
soit 1< D2/λL = N.

3.2.1. Les relations écrites traduisent le principe d’Huyghens Fresnel.
Chaque point réémet une onde sphérique dont l’amplitude et la phase dépendent
de l’onde incidente.
Le déphasage j = exp (j. π/2) correspond au passage par un foyer.
3.2.2. Soit la projection P1 sur oz
(P1M)2 = [P1H1 + H1H + HM]2
(P1M)2 = (P1H1)2 + (H1H)2 + (HM)2 - 2 H1P1. HM
(P1M)2 = R’1)2 + (H1H)2 - 2 R’1. R.
Le point P1 appartient au cercle de rayon L,
donc
(H1P1)2 + (Z - L)2 = L2 Z cote du point P1
comme D << L, alors Z << L.
d’où Z ≈ (R’1)2/2L.
P1M2 = R2 + z2 + (R’1)2 [1 - z/L]
P1M = z + [R2 - 2.R. R’1 + R’12 (1- z/L)] /2z
on peut donc écrire :
(1/P1M) (exp jk. P1M) = (1/z) (exp jk z) exp (jk/2z) [R2+(R’1)2 (1-z/L) -2R.R’1
Le principe d’Huyghens entraîne alors l’expression donnée dans l’énoncé.
3.2.3. Evaluons U (P2).
U (P2) = Aγ2/L ∫∫s U(P1) K (P1P2) ds.
or K(P1P2) = exp jk/2L [R’21 - 2R’1.R’2]
car z ≈ L et R ≈ R’2
K (P1P2) ≈ exp - (jk/L) R’1.R’2 = exp - j k/L [x1x2 + y1y2)
d’où U (P2) = Aγ2/L ∫∫s1 [K1 exp [- π(R’1)2/D21] exp - (jk/L) (x1x2+y1y2) dx, dy2
car ds ≈ dx1 dy1.
U (P2) = Aγ2K1/ J (x2). J (y2).
avec J (x2) = ∫x1 exp [- π(x21)/D12 - j k/L (x1x2)]. dx1
à l’intervalle de variation de x1 étant étendue à l’∞.
J(x2) = ∫ exp [- (π/D12 )x12 - jk x2x1/L] dx1
J(x2) = ∫ exp - π/D12 [x1 + jkD12x2/L.2 π]2 . exp - k2D21x22//4πL2 . dx1

d’où J (x2) = exp (- k2d12x22D1/4.π.L2)
d’où U (P2) =Aγ2K1L-1.D21 exp (-k2D12 [x22 + y22]/4. π.2L2)
U (P2) = Aγ2K1L-1.D21 exp - (k2D12.R’2/4. π.L2)
or U (P2) = K2 exp [-π (R’2)2/D22]
d’où k2D12/4. π.L2 = π/D22 soit D1D2 = λL
(D21.A γ2K1) = K2.L
Par permutation des indices on obtient en fait, une troisième relation :
(D22. A γ1K2) = K1.L
il vient γ1. γ2. (A. λ)2 = 1.
3.2.4.
A = J/λ alors γ1. γ2 = - 1
donc exp - 2j.k.L = - 1. alors 2J.kL = (2m+1) π
k = π (2m + 1)/2L + ωm/c
ωm = (2m+1) .πc/2L
En reprenant les relations on obtient
K2 = (-1)m K1D1/D2
soit une cavité oscillant à la pulsation xo moyenne.
ωo - Δω/2 < ωo + Δω/2
Le nombre de mode N dans la cavité est égal au nombre d’entiers satisfaisant à
ω = (2m+1) π c/2L dans la bande de pulsation considérée.
Δω = Δm πc/2L soit N = Δm = 2L Δω/πc
soit N = 2L Δλ/.λ.2
Dans le visible, si on prend L = 1µm comme pour la cavité cubique
Δλ = 400 nm λ = 600 nm
N = 2 modes alors que l’on avait 100 modes pour la cavité cubique.
La cavité confocale est donc plus sélective.

Concours Physique ENSI 1994 (Corrigé)

PREMIERE PARTIE
1 - Amplificateur sélectif à circuit résonant
1 - Etude du bobinage primaire.
1 - 1 - a - Les lignes de champ sont des cercles dont les axes sont confondus avec l'axe Oz. En effet tout plan contenant l'axe Oz est plan de symétrie de la distribution et$\overrightarrow B $ est normal à chacun de ces plans.
1 - 1 - b - Prenons le contour fermé suivant :
- un arc de cercle de rayon${r_0}$ et d'angle passant par O'
- un arc de cercle de rayon${r_0} + r\cos \theta $ et d'angle passant par P - on ferme naturellement le contour (la circulation y est nulle).
Le théorème d'Ampère s'exprime :$\overrightarrow B \overrightarrow {dl} = 0$
soit ${B_0}.\frac{{{r_0}\alpha }}{{2\pi }} - {B_P} + r\cos \theta \frac{\alpha }{{2\pi }} = 0$ou encore, avec${B_0} = \frac{{{n_1}{\mu _0}I}}{{2\pi {r_0}}}$
${B_P} = \frac{{{B_0}}}{{1 + \frac{r}{{{r_0}}}\cos \theta }}$
1 - 1 - c - En calculant le flux$\Phi $ engendré par le tore sur lui-même :
$\Phi = {n_1}.\overrightarrow B d\overrightarrow S = \frac{{n_1^2{\mu _0}I}}{{2\pi {r_0}\left( {1 + \frac{r}{{{r_0}}}\cos \theta } \right)}}r{\rm{ }}dr{\rm{ d}}\theta $ comme${r_0} > > a \approx r$
$\Phi = \frac{{n_1^2{\mu _0}I}}{{2\pi {r_0}}}\left( {1 - \frac{r}{{{r_0}}}\cos \theta + \frac{r}{{{r_0}}}\cos \theta } \right)r{\rm{ }}dr{\rm{ }}d\theta $
on obtient finalement pour L après simplification :
$L = \frac{{n_1^2{\mu _0}{a^2}}}{{2{r_0}}}\left( {1 + \frac{{{a^2}}}{{4r_0^2}}} \right)$

1 - 1 - d - Comme on a$L = {L_1}\left( {1 + \frac{{{a^2}}}{{4r_0^2}}} \right)$,$L \approx {L_1}$avec une précision inférieure à 1% si $\frac{{{a^2}}}{{4r_0^2}} < 0,01{\text{ soit }}\frac{a}{{{r_0}}} < 0,2$.
1 - 1 - e - La résistance de la bobine est donnée par la relation suivante :
$R = \rho \frac{I}{s}{\text{ avec }}I = {n_1}2\pi a{\text{, }}s = \pi \frac{{{d^2}}}{4}{\text{ et }}{n_1} = 2\pi \left( {{r_0} - \frac{a}{d}} \right)$
Soit$R = \frac{{16\pi \rho \left( {{r_0} - a} \right)}}{{{d^3}}}$ et$\frac{{{L_1}}}{R} = \frac{{{\mu _0}\pi ad({r_0} - a)}}{{8\rho {r_0}}}$.
1 - 1 - f - A.N.${L_1} = 0,11mH.{\rm{ }}\frac{{{L_1}}}{R} = {5,92.10^{ - 5}}H.{\Omega ^{ - 1}}.{\rm{ }}\frac{{{L_1}{\omega _0}}}{R} = 37,2$
2 - Etude du bobinage secondaire
1 - 2 - a -${\Phi _{1 \to 2}} = {n_2}.\overrightarrow {{B_1}} .d\overrightarrow {{S_2}} = M.{I_1}$ soit, en négligeant les temes du second ordre$M = \frac{{{n_1}{n_2}{\mu _0}{a^2}}}{{2{r_0}}}$.
1 - 2 - b -$M = \sqrt {{L_1}.{L_2}} $.
3 - Etude de l'amplificateur
1 - 3 - a - la puisanace complexe est :
$\underline P = \frac{1}{2}\underline {{Z_e}} \underline {{I_{\max }}} \underline {{{\dot I}_{\max }}} = \frac{1}{2}\underline {{Z_e}} {\left| {\underline {{I_{\max }}} } \right|^2} = \underline {{Z_e}} {\left| {\underline {{I_0}} } \right|^2}{\text{ avec }}P = {\mathop{\rm Re}\nolimits} (\underline P )$.
${Z_e} = R//{C_1}//{L_1} = \frac{{j{R_1}{L_1}\omega }}{{{R_1}\left( {1 - {L_1}{C_1}{\omega ^2} + j{L_1}\omega } \right)}}$soit$\underline P = \frac{{j{R_1}{L_1}\omega .I_0^2}}{{{R_1}\left( {1 - \frac{{{\omega ^2}}}{{\omega _0^2}} + j{L_1}\omega } \right)}}$
finalement
$P = \frac{{{R_1}I_0^2}}{{1 + \frac{{R_1^2}}{{L_1^2\omega _0^2}}{{\left( {x - \frac{1}{x}} \right)}^2}}}{\text{ avec }}{P_{\max }} = {R_1}I_0^1{\text{ et }}Q = \frac{{{R_1}}}{{{L_1}{\omega _0}}}$
1 - 3 - b -$\frac{{{P_{\max }}}}{{1 + {Q^2}{{\left( {x - \frac{1}{x}} \right)}^2}}} = \frac{{{P_{\max }}}}{2}$ soit$x - \frac{1}{x} = \pm \frac{1}{Q}$ (deux équations du second degré)
( 1 )${x_{1,3}} = \frac{1}{2} - \frac{1}{Q} \pm \sqrt {{{\left( {\frac{1}{Q}} \right)}^2} + 4} {\text{ seule la racine positive est accebtable }}({x_1}).$
( 2 )${x_{2,4}} = \frac{1}{2}\frac{1}{Q} \pm \sqrt {{{\left( {\frac{1}{Q}} \right)}^2} + 4} {\text{ seule la racine positive est acceptable }}({x_2}).$
$\Delta {\omega _0} = \left( {{x_2} - {x_1}} \right){\omega _0} = \frac{{{\omega _0}}}{Q}$soit $\frac{{{\omega _0}}}{{\Delta {\omega _0}}} = Q$
1 - 3 - c -$\underline {{V_s}} = \underline {{Z_e}} \underline {{I_0}} = \underline {{Z_e}} s\underline {{V_e}} {\text{ soit }}G = \left| {\underline {{Z_e}} } \right|s{\text{ finalement }}G = \frac{{{R_1}.s}}{{1 + {Q^2}{{\left( {x - \frac{1}{x}} \right)}^2}}}$
${V_{s,\max }}{\text{ est donné pour }}x = 1{\rm{ }}{V_{s,\max }} = {R_1}{I_0}$
${V_{s,\min }}{\text{ est donné pour }}{\omega _1}{\text{ ou }}{\omega _2}{\text{ soit }}{V_{s,\min }} = \frac{{{R_1}{I_0}}}{{\sqrt 2 }}$
1 - 3 - d - ${V_e} = {A_0} + \sum\limits_{n = 1}^\infty {{C_n}\cos (n{\omega _0}t) + {\psi _n}} $
${V_{n,s}} = G(n{\omega _0}).{V_{n,e}}{\text{ on peut donc d\'e finir }}{\tau _{n,e}} = \frac{{{C_{n,e}}}}{{{C_{1,e}}}}{\text{ et }}{\tau _{n,s}} = \frac{{G(n{\omega _0}).{C_{n,e}}}}{{G({\omega _0}).{C_{1,e}}}}$
On obtient pour l'atténuation en décibels
${\delta _n} = \frac{{{\tau _{n,e}}}}{{{\tau _{n,s}}}} = \frac{{G(n{\omega _0})}}{{G({\omega _0})}} = \frac{{1 + {Q^2}{{\left( {x - \frac{1}{x}} \right)}^2}}}{{1 + {Q^2}{{\left( {nx - \frac{1}{{nx}}} \right)}^2}}}$ comme x= 1 et n = 2,${\delta _2} = \frac{1}{{1 + {Q^2}{{\left( {\frac{3}{2}} \right)}^2}}}$
Tout calculs fait on trouve$Q = 2.$
1 - 3 - e – Posons${R_{eq}} = {R_1}//{R_u}$ On exprime alors Q' par $Q' = \frac{{{R_{eq}}}}{{{L_1}{\omega _0}}}$ soit $Q' = \frac{{Q{R_u}}}{{{R_1} + {R_u}}}$ donc $\Delta \omega _0^{'}\frac{{{R_1} + {R_u}}}{{{R_u}}}$
Si ${R_u} = {R_1}$ alors $\Delta \omega _0^{'} = 2.\Delta {\omega _0}$

4 - Couplage magnétique de l'amplificateur à une charge
1 - 4 - a - ( 1 ) $\underline {{V_1}} = j{L_1}\omega \underline {{I_1}} + jM\omega \left( {\underline {{I_1}} + \underline {{I_2}} } \right) + j{L_2}\omega \left( {\underline {{I_1}} + \underline {{I_2}} } \right) + jM\omega \underline {{I_1}} $
( 2 ) $\underline {{V_2}} = jM\omega \underline {{I_1}} + j{L_2}\omega \left( {\underline {{I_1}} + \underline {{I_2}} } \right)$
( 3 ) $\underline {{V_2}} = - {R_u}\underline {{I_2}} $
De ( 2 ) et ( 3 ) on tire ( 4 ) $\underline {{I_2}} = \frac{{j{L_2}\omega + jM\omega }}{{ - {R_u} + j{L_2}\omega }}\underline {{I_1}} $
En substituant ( 4 ) dans ( 2 ) et remarquant que$\frac{{{L_1}}}{{{L_2}}} = \frac{{n_1^2}}{{n_2^2}}{\text{ et que }}\frac{{2M}}{{{L_2}}} = \frac{{2{n_1}{n_2}}}{{n_2^2}}$
on trouve$\frac{{\underline {{V_1}} }}{{\underline {{I_1}} }} = \underline {{Z_e}} = {\mu ^2}\frac{{j{L_2}\omega {R_u}}}{{{R_u} + j{L_2}\omega }}{\text{ avec }}\mu = \frac{{{n_1} + {n_2}}}{{{n_2}}}$.
1 - 4 - b - Calculons la pulsation de résonance du montage.
Comme on nous demande dans la question - c - le facteur de qualité Q, exprimons la puissance moyenne absorbée par l'impédance constituée${R_1},{\rm{ }}{C_2}{\text{, les bobines coupl\'e es et }}{R_u}$. En premier lieu, exprimons l'impédance équivalente$\underline {{Z_{eq}}} = {R_1}//{C_2}//\underline {{Z_e}} $ soit
$\underline {{Z_{eq}}} = \frac{{j{R_1}{R_u}{\mu ^2}{L_2}\omega }}{{{R_1}{R_u}\left( {1 - {L_2}{C_2}{\mu ^2}{\omega ^2}} \right) + j{L_2}\omega \left( {{R_1} + {\mu ^2}{R_u}} \right)}}$
Comme$P = {\mathop{\rm Re}\nolimits} \left( {\underline {{Z_e}} } \right){\left| {\underline {{I_0}} } \right|^2}$ on trouve alors tout calcul fait
$P = \frac{{\frac{{{R_1}{R_u}{\mu ^2}}}{{{R_1} + {\mu ^2}{R_u}}}I_0^2}}{{1 + {{\left( {\frac{{{R_1}{R_u}}}{{{R_1} + {\mu ^2}{R_u}}}} \right)}^2}\frac{1}{{L_2^2\omega _0^2}}{{\left( {\frac{1}{x} - x} \right)}^2}}}$ soit${P_{\max }} = \frac{{{R_1}{R_u}{\mu ^2}}}{{{R_1} + {\mu ^2}{R_u}}}I_0^2{\rm{ }}\omega _0^2 = \frac{1}{{{L_2}{C_2}{\mu ^2}}}$
Et$Q = \frac{{{R_1}{R_u}}}{{{R_1} + {\mu ^2}{R_u}}}\frac{1}{{{L_2}{\omega _0}}}$.
On veut$\frac{1}{{{L_2}{C_2}{\mu ^2}}} = \frac{1}{{{L_1}{C_1}}}$soit${C_2} = \frac{{n_1^2}}{{{{({n_1} + {n_2})}^2}}}{C_1}$
1 - 4 - c - En définissant$\Delta Q = Q - Q'$avec$Q = \frac{{{R_1}}}{{{\mu ^2}{L_2}{\omega _0}}}$ et$Q' = \frac{{{R_1}{R_u}}}{{\left( {{R_1} + {\mu ^2}{R_u}} \right){L_2}{\omega _0}}}$
on obtient$\frac{{\Delta Q}}{Q} = \frac{1}{{1 + {\mu ^2}}}$. A..N. = 9,9
DEUXIEME PARTIE
11 - Exemples de filtres actifs à circuits RC
11 - 1 - Expression générale de la fonction de transfert d'un filtre passe -bande du second ordre.
Calculons le module de$\underline {T(j\omega )}$ .\[\left| {\underline {T(j\omega )} } \right| = \frac{{{G_0}}}{{{{\left( {1 + \frac{1}{{4{\alpha ^2}}}{{\left( {\frac{1}{x} - x} \right)}^2}} \right)}^{1/2}}}}.\] Soit F la fonction suivante$F = 1 + \frac{1}{{4{\alpha ^2}}}{\left( {\frac{1}{x} - x} \right)^2}$, étudions la sur l'intervalle$\left[ {0, + \infty } \right]$.
$\frac{{dF}}{{dx}} = \frac{{ - 1}}{{2{\alpha ^2}}}\left( {\frac{1}{x} - x} \right)\left( {\frac{1}{{{x^2}}} + 1} \right)$
soit
x     0            1                                  $ + \infty $
F      ↘     min           ↗                 $ + \infty $
F'       -      0              +
T 0   ↗     ${G_0}$            ↘                    0
La forme de la courbe permet de conclure sur la nature du filtre.
Calculons la bande passante :$\frac{{{G_0}}}{{\sqrt 2 }} = \frac{{{G_0}}}{{\sqrt {\left( {\frac{1}{x} - x} \right)} \left( {\frac{1}{{4{\alpha ^2}}} + 1} \right)}}$ soit$x - \frac{1}{x} = \pm 2\alpha $(deux équations du second degré)
( 1 )${x_{1,3}} = - \alpha \pm \sqrt {{\alpha ^2} + 1} $ seule${x_1}$est positive.
( 2 )${x_{2,4}} = \alpha \pm \sqrt {{\alpha ^2} + 1} $ seule${x_2}$est positive.
$\Delta {\omega _0} = \left( {{x_2} - {x_1}} \right){\omega _0} = 2\alpha {\omega _0}$ soit$Q = \frac{{{\omega _0}}}{{\Delta {\omega _0}}} = \frac{1}{{2{\alpha ^2}}}$

11 - 2 - Filtre actif à structure de Sallen et Key
11 - 2 - a - ( 1 ) $\underline {{V^ - }} = \underline {{V^ + }} = \frac{1}{k}\underline {{V_s}} $
( 2 )$\underline {{V_A}} = \frac{{\underline {{Y_1}} \underline {{V_e}} + \underline {{Y_2}} \underline {{V^ + }} + \underline {{Y_3}} \underline {{V_s}} }}{{\underline {{Y_1}} + \underline {{Y_2}} + \underline {{Y_3}} }}$
( 3 )$\underline {{V_A}} = \underline {{V^ + }} \frac{{\underline {{Y_2}} + \underline {{Y_4}} }}{{\underline {{Y_2}} }}$
De ( 1 ) et ( 3 ) on tire ( 4 )$\underline {{V_A}} = \underline {{V_s}} \frac{{\underline {{Y_2}} + \underline {{Y_4}} }}{{\underline {{Y_2}} .k}}$
En égalisant ( 2 ) et ( 4 ) on trouve$\frac{{\underline {{V_s}} }}{{\underline {{V_e}} }} = \frac{{\underline {{Y_1}} \underline {{Y_2}} .k}}{{\left( {\underline {{Y_2}} + \underline {{Y_4}} } \right)\left( {\underline {{Y_1}} + \underline {{Y_2}} + \underline {{Y_3}} } \right) - \underline {Y_2^2} - \underline {{Y_2}} \underline {{Y_3}} .k}}$
11 - 2 - b – Avec$\underline {{Y_1}} = \frac{1}{R}$,$\underline {{Y_2}} = jC\omega $,$\underline {{Y_3}} = \frac{1}{R}$,$\underline {{Y_4}} = \frac{1}{R} + jC\omega $et en exprimant $\underline {T\left( {j\omega } \right)} = \frac{{{G_0}}}{{1 + \frac{1}{{2\alpha jx}} + \frac{{jx}}{{2\alpha }}}}$ on trouve tout calcul fait $\underline {T\left( {j\omega } \right)} = \frac{k}{{\left( {5 - k} \right)\left( {1 + \frac{2}{{jRC\omega \left( {5 - k} \right)}} + \frac{{jRC\omega }}{{5 - k}}} \right)}}$
soit en identifiant: ( 1 )${G_0} = \frac{{k - 1}}{{5 - k}}$
( 2 )$\frac{{jx}}{{2\alpha }} = \frac{{jRC\omega }}{{5 - k}}$
( 3 )$\frac{1}{{jx2\alpha }} = \frac{2}{{jRC\omega \left( {5 - k} \right)}}$
En effectuant le produit ( 1 )*( 2 ) on tire$\alpha = \frac{{5 - k}}{{2\sqrt 2 }}$ on trouve alors pour${\omega _0} = \frac{{\sqrt 2 }}{{RC}}$ et $Q = \frac{{\sqrt 2 }}{{5 - k}}$
11 - 2 - c - A partir de l'équation différentielle homogène associée au montage, étudions la stabilité du système :$v(t) + \frac{{2\alpha }}{{{\omega _0}}}\frac{{dv(t)}}{{dt}} + \frac{1}{{\omega _0^2}}\frac{{{d^2}v(t)}}{{d{t^2}}} = 0$
le discriminant réduit associé à l'équation caractéristique de cette équation différentielle s'exprime ainsi$\Delta ' = \frac{1}{{\omega _0^2}}\left( {{\alpha ^2} - 1} \right)$.
Si k = 5 alors = 0 et le sytéme oscille.
Si k > 5 alors < 0. On peut distinguer deux cas :$\left| \alpha \right| < 1{\text{ et }}\left| \alpha \right| > 1$.
Dans ces deux cas le systéme est instable.
11 - 3 - Filtre actif à varialble d'état
11 - 3 - a - $\underline {{V_A}} = - r\underline {{I_1}} $et$\underline {{V_A}} = - 2r\underline {{I_1}} + \underline {{V_s}} $ soit$\underline {{V_A}} = - \underline {{V_s}} $
De plus$\underline {{V_A}} = R\underline {{I_1}} - \underline {{V_s}} $ et$\underline {{V_0}} = - \frac{1}{{jC\omega }}\underline {{I_1}} $ soit$\frac{{\underline {{V_0}} }}{{{V_s}}} = \frac{1}{{jRC\omega }}$ ( 1 )
En écrivant la loi des noeuds en D on obtient$ - \frac{{\underline {{V_s}} }}{{\underline {{Z_{eq}}} }} - \frac{{\underline {{V_e}} }}{{\underline {{R_2}} }} = \frac{{\underline {{V_0}} }}{{\underline R }}$ ( 2 )
En combinant ( 1 ) et ( 2 ) on obtient la relation demandé sous la forme
$\underline {T\left( {j\omega } \right)} = \frac{{{G_0}}}{{1 + \frac{1}{{2\alpha jx}} + \frac{{jx}}{{2\alpha }}}} = - \frac{{{R_1}}}{{{R_2}}}\frac{1}{{1 + \frac{{{R_1}}}{{j{R^2}C\omega }} + j{R_1}C\omega }}$
Soit${G_0} = - \frac{{{R_1}}}{{{R_2}}}{\rm{ }}{\text{, }}\alpha = \frac{R}{{2{R_1}}}{\rm{ }}{\text{, }}{\omega _0} = \frac{1}{{RC}}{\text{ et }}Q = \frac{{{R_1}}}{R}$
11 - 4 - Filtres actifs à condensateurs commutés
11 - 4 - a -${e_1} = u(t) + rC\frac{{du(t)}}{{dt}}$ soit$u(t) = {e_1} + Cte.\exp \left( { - \frac{t}{\tau }} \right)$. Le temps de charge étant trés faible par rapport à la période du système, on peut facilement supposer que la tension aux bornes du condensateur à t = 0 est égale à${e_2}$.
La constante se déduit facilement$Cte = {e_2} - {e_1}$. La tension aux bornes du condensateur pour l'intervalle de temps$\left[ {n{T_c},n + \frac{1}{2}{T_c}} \right]$ est ${u_1}(t) = {e_1} + \left( {{e_2} - {e_1}} \right)\exp \left( { - \frac{t}{\tau }} \right)$.
De même on trouve pour l'intervalle de temps$\left[ {n + \frac{1}{2}{T_c},\left( {n + 1} \right){T_c}} \right]$
${u_2}(t) = {e_1} + \left( {{e_1} - {e_2}} \right)\exp \left( { - \frac{{t - \frac{{{T_c}}}{2}}}{\tau }} \right)$
11 - 4 - b - ${i_1}(t) = C\frac{{d{u_1}(t)}}{{dt}} = \frac{C}{\tau }\left( {{e_1} - {e_2}} \right)\exp \left( { - \frac{t}{\tau }} \right)$ de même pour${i_2}(t)$
${i_2}(t) = C\frac{{d{u_1}(t)}}{{dt}} = \frac{C}{\tau }\left( {{e_2} - {e_1}} \right)\exp \left( { - \frac{t}{\tau }} \right)$
La valeur moyenne étant définit ainsi$\left\langle {{i_1}(t)} \right\rangle = \frac{1}{{{T_c}}}\int_0^{{T_c}} {{i_1}(t)dt} $.
L'intégrale donne, en tenant compte de l'approximation$\tau < < {T_c}{\rm{ }}\left\langle {{i_1}(t)} \right\rangle = \frac{C}{{{T_c}}}\left( {{e_1} - {e_2}} \right)$.
De même pour$\left\langle {{i_2}(t)} \right\rangle $ on obtient $\left\langle {{i_2}(t)} \right\rangle = \frac{C}{{{T_c}}}\left( {{e_2} - {e_1}} \right) = - \left\langle {{i_1}(t)} \right\rangle $

11 - 4 - c - $\left\langle {{u_{AB}}} \right\rangle = {e_1} - {e_2} = \left\langle {{i_{AB}}(t)} \right\rangle {R_e}$avec
$\left\langle {{i_{AB}}(t)} \right\rangle = \left\langle {{i_1}(t) - {i_2}(t)} \right\rangle = \left\langle {{i_1}(t)} \right\rangle - \left\langle {{i_2}(t)} \right\rangle = 2\left\langle {{i_1}(t)} \right\rangle = \frac{{2C}}{{{T_c}}}\left( {{e_1} - {e_2}} \right)$
Soit${R_e} = \frac{{{T_c}}}{{2C}} = \frac{1}{{2C{f_c}}}$
11 - 4 - d - Comme la fréquence de commutation de l'intérupteur est bien suppérieure à la fréquence du signal, pour un intervalle de temps de l'ordre de${T_c}$, le signal apparait continu pour l'intérruteur lors de plusieurs commutations.
11 - 4 - e - En utilisant le théorème de Millman au point A :
$\underline {{V_A}} = \frac{{\underline {{Y_1}} \underline {{V_e}} + \underline {{Y_3}} \underline {{V_s}} }}{{\underline {{Y_1}} + \underline {{Y_4}} + \underline {{Y_3}} }}$
et au point B$\underline {{V_B}} = 0 = \underline {{Y_2}} \underline {{V_s}} + \underline {{Y_4}} \underline {{V_A}} $. En combinant les deux équations
précédentes , en fonction des admittances$\frac{{\underline {{V_s}} }}{{\underline {{V_e}} }} = \frac{{\underline {{Y_1}} }}{{ - \frac{{\underline {{Y_2}} }}{{\underline {{Y_4}} }}\left( {\underline {{Y_1}} + \underline {{Y_4}} + \underline {{Y_3}} } \right) - \underline {{Y_3}} }}$soit , en exprimant ce résultat en fonction des admittaces :$\underline {{Y_1}} = \frac{1}{{{R_e}}} = \underline {{Y_2}} {\text{, }}\underline {{Y_2}} = j{C_2}\omega {\text{, }}\underline {{Y_4}} = j{C_1}\omega $ et sous la forme déja vue :$\frac{{\underline {{V_s}} }}{{{V_e}}} = \frac{{ - 1}}{{\left( {1 + \frac{{{C_2}}}{{{C_1}}}} \right)\left( {1 + \frac{{{C_1}}}{{j{R_e}{C_1}\omega \left( {{C_1} + {C_2}} \right)}} + \frac{{j{R_e}{C_1}{C_2}\omega }}{{{C_1} + {C_2}}}} \right)}}$.
On peut exprimer les grandeurs demandées :${G_0} = \frac{{ - {C_1}}}{{{C_1} + {C_2}}}{\text{, }}\alpha = \frac{{{C_1} + {C_2}}}{{2\sqrt {{C_1}{C_2}} }}{\text{, }}{\omega _0} = \frac{1}{{{R_e}\sqrt {{C_1}{C_2}} }} = \frac{{2C{f_c}}}{{\sqrt {{C_1}{C_2}} }}{\text{ et }}Q = \frac{{\sqrt {{C_1}{C_2}} }}{{{C_1} + {C_2}}}$

Concours Physique Concours Commun TPE 1994 (Corrigé)

Corrigé épreuve physique commune - T.P.E 1994
PREMIER PROBLÈME - ÉLECTRONIQUE
I Référence de tension à diode Zener
I-1)
La loi des mailles et la loi des nœuds fournissent tout d'abord:
$\left\{ \begin{array}{l}{V_1} = {V_2} + R{I_1}\\U = - {V_2}\\{I_1} + I = {I_2}\end{array} \right.$
Il est possible d'obtenir V2 = V1 -R(I2 -I) = f(V1,I2) à condition d'éliminer I. On distingue alors suivant la valeur de U
  • U > Vd ⇔ Dz est en mode passant (Zone 1 de la figure 1)
${{\text{V}}_{\text{2}}}=\frac{{{R}_{d}}}{R+{{R}_{d}}}{{V}_{1}}-\frac{R{{R}_{d}}}{R+{{R}_{d}}}{{I}_{2}}+\frac{R{{V}_{d}}}{R+{{R}_{d}}}\text{ Si }{{\text{V}}_{\text{2}}}<-{{V}_{d}}$
$I = \frac{{U - {V_d}}}{{{R_d}}} = \frac{{ - {V_2} - {V_d}}}{{{R_d}}}$ $ \Rightarrow {\rm{ }}{{\rm{V}}_{\rm{2}}} = {V_1} - \frac{R}{{{R_d}}}({R_d}{I_2} + {V_2} + {V_d})$ d'où le résultat:
  • -Vz < U < Vd ⇔ Dz est en mode bloqué (Zone 2 de la figure 1) alors I = 0
d'où le résultat:
$\Rightarrow \text{ }{{\text{V}}_{\text{2}}}={{V}_{1}}-R{{I}_{2}}\text{ si }{{\text{V}}_{\text{2 }}}<\text{ -}{{\text{V}}_{\text{d}}}$
  • U <- Vz ⇔ Dz est en mode passant inverse (Zone 3 de la figure 1)
$I = \frac{{U + {V_z}}}{{{R_z}}} = \frac{{ - {V_2} + {V_z}}}{{{R_z}}}$ $ \Rightarrow {\rm{ }}{{\rm{V}}_{\rm{2}}} = {V_1} - \frac{R}{{{R_z}}}({R_z}{I_2} + {V_2} - {V_z})$ d'où le résultat:
$\Rightarrow \text{ }{{\text{V}}_{\text{2}}}={{V}_{1}}-\frac{R}{{{R}_{z}}}({{R}_{z}}{{I}_{2}}+{{V}_{2}}-{{V}_{z}})$

I-2)
On en déduit que la caractéristique V2 = f(I2) à V1 Cte est une fonction affine par morceaux
Zone 1:pente= ${\rm{ }}\frac{{{\rm{ - R}}{{\rm{R}}_{\rm{d}}}}}{{R + {R_d}}} \approx - {R_d}$
Zone 2:pente= ${\rm{ }} - R$
Zone 3:pente= ${\rm{ }}\frac{{{\rm{ - R}}{{\rm{R}}_{\rm{z}}}}}{{R + {R_z}}} \approx - {R_z}$
La zone 3 ayant une pente quasi nulle représente le domaine de régulation V2 ≈Vz
Lorsque le point de fonctionnement se trouve dans la zone 1 le courant I2 et la tension V2 sont de signes contraires, ce qui veut dire que l'on a affaire à une charge constituée par un générateur. Les rôles de l'entrée et de la sortie sont inversés.
I-3)
Cette fois les pentes sont:
Zone 1: pente = $\frac{{{{\rm{R}}_{\rm{z}}}}}{{R + {R_z}}} \approx \frac{{{{\rm{R}}_{\rm{z}}}}}{R}$
Zone 2: pente = 1
Zone 3: pente = $\frac{{{{\rm{R}}_{\rm{d}}}}}{{R + {R_d}}} \approx \frac{{{{\rm{R}}_{\rm{d}}}}}{R}$
La zone 3 ayant une pente quasi nulle représente le domaine de régulation V2 ≈Vz
I-4)Dans la zone 3 on a établi que : ${V_2} = \frac{{{R_z}}}{{R + {R_z}}}{V_1} - \frac{{R{R_z}}}{{R + {R_z}}}{I_2} + \frac{R}{{R + {R_z}}}{V_z}$
Donc les variations sont données par: $\Delta {V_2} = \frac{{{R_z}}}{{R + {R_z}}}\Delta {V_1} - \frac{{R{R_z}}}{{R + {R_z}}}\Delta {I_2} = S\Delta {V_1} - \rho \Delta {I_2}$
$S=\frac{{{R}_{z}}}{R+{{R}_{z}}}\approx \frac{{{\text{R}}_{\text{z}}}}{\text{R}}\text{ et }\rho \text{=}\frac{R{{R}_{z}}}{R+{{R}_{z}}}\approx {{R}_{z}}$
II Fonctions à seuil
II-1) L'A-O est en régime linéaire donc V- est nul. Alors puisque E1 est positive, I1 est positif
II-1-a)Si VS est positif (strictement) alors le courant I2 est négatif et I0 est positif (strictement).
Alors i1 =I1 -I2 est positif, donc la diode D1 est passante,
Alors VK = V- = 0 comme VS,
Alors la diode D2 est traversée par i2 positif ou nul ⇒ I2 =I0 +i2 est positif
Il y a donc contradiction logique et VS ne peut pas être strictement positf. II-1-b)Si VS est négatif (strictement) alors le courant I2 est positif et I0 est négatif (strictement).
Alors i2 =I0 -I2 est négatif strictement, mais i2 ne peut être que positif ou nul donc nouvelle contradiction logique et VS ne peut pas être strictement négatif. II-1-c) VS ne peut être que nul (D2 est au repos, D1 traversée par I1 court-circuite R2 ).

II-2) V- est toujours nul. Et puisque E1 est négative, I1 est négatif.
• Si VS est négatif ou nul alors le courant I2 est positif ou nul.
Alors i1 =I1 -I2 est négatif, ce qui n'est pas possible,
VS ne peut pas être strictement négatif ou nul.
• Si VS est positif (strictement) alors le courant I2 est négatif, I0 est positif
Alors i2 =I0 -I2 est positif (strictement), la diode D2 est passante.
Alors VK = VS > 0 la diode D1 est bloquée et on a I1 = I2 ${\rm{ soit }}\frac{{{{\rm{E}}_{\rm{1}}}}}{{{{\rm{R}}_{\rm{1}}}}} = - \frac{{{V_S}}}{{{R_2}}}$ • La seule situation possible est: ${\rm{ }}{{\rm{V}}_{\rm{S}}} = - \frac{{{{\rm{R}}_{\rm{2}}}}}{{{{\rm{R}}_{\rm{1}}}}}{E_1}{\rm{ }}$
II-3)
La caractéristique VS = f(E1) est donnée ci-contre :
C'est celle d'un ampli-inverseur qui au delà d'un certain seuil (zéro ici) est coupé.
II-4)
II-4a) Plusieurs solutions sont possibles pour placer les sources dont les valeurs sont algébriques.
Un schéma classique est représenté ci-contre et en absence d'autres défautson peut dire que l'on a ε = 0
II-4b) Pour un A-O 741 on a typiquement:
Ed de l'ordre de quelques mV
Ip de l'ordre de quelques nA
II-4c)On admet que les défauts ne modifient pas l'état prévu des diodes. Pour E1 > 0 on a D1 passante et D2 bloquée. D'où le schéma:
$\begin{array}{l}{I_2} = {I_0}{\rm{ }} \Rightarrow {\rm{ }}\\\frac{{{{\rm{E}}_{\rm{d}}} - {V_S}}}{{{R_2}}} = \frac{{{V_S}}}{{{R_L}}}\\{V_S} = \frac{{{R_L}}}{{{R_2} + {R_L}}}{E_d}\end{array}$
Mais D2 bloquée ⇒
V- -VS = Ed - VS < 0 impose que Ed <0
Sinon D2 passante est alors VS = Ed
II-5)VS = µε avec ε = V- - V+ = VA Mais l'amplificateur n'ayant pour seul défaut qu'un gain fini on peut écrire, d'après le théorème de Millman:
${{\rm{V}}_{\rm{A}}}\left( {\frac{1}{{{R_1}}} + \frac{1}{{{R_2}}}} \right) = \frac{E}{{{R_1}}} + \frac{{{V_S}}}{{{R_2}}}{\rm{ }} \Rightarrow {\rm{ }} - \frac{{{{\rm{V}}_{\rm{S}}}}}{\mu }\left( {\frac{1}{{{R_1}}} + \frac{1}{{{R_2}}}} \right) = \frac{E}{{{R_1}}} + \frac{{{V_S}}}{{{R_2}}}{\rm{ }}$
\[{{V}_{S}}=\left( \frac{1}{\mu {{R}_{1}}}+\frac{1}{\mu {{R}_{2}}}+\frac{1}{{{R}_{2}}} \right)=-\frac{E}{{{R}_{1}}}\Rightarrow \frac{{{V}_{S}}}{E}=\frac{-1}{{{R}_{1}}\left( \frac{1}{\mu {{R}_{1}}}+\frac{1}{\mu {{R}_{2}}}+\frac{1}{{{R}_{2}}} \right)}=\frac{-{{R}_{2}}/{{R}_{1}}}{1+\frac{1}{\mu }\left( \frac{{{R}_{1}}+{{R}_{2}}}{{{R}_{1}}} \right)}\] Le montage est donc celui d'un ampli-inverseur idéal rétroactionné par un opérateur de retour β vérifiant: $\beta .(gain{\rm{ ideal) = }}\frac{{\rm{1}}}{{\mu {\rm{b}}}}$ Or on sait qu'une rétroaction diminue le gain de l'opérateur direct. C'est bien vérifié ici.
II-6)
Si les défauts ne modifient pas l'état prévu des diodes. Pour E1<0 D2 est passante, D1 est bloquée. Le théorème de Millman donne:
$\begin{array}{l}{\rm{ }}{V^ - } = - \varepsilon = \frac{{\frac{{{E_1}}}{{{R_1}}} + \frac{{{V_S}}}{{{R_2}}}}}{{\frac{1}{{{R_1}}} + \frac{1}{{{R_2}}}}}\\{\rm{et }}\mu \varepsilon = {U_2} + {V_S}\end{array}$
On peut éliminer ε et avoir VS en fonction de g(i2 )=U2
On obtient ainsi:
\[-{{V}_{S}}\left( \frac{1+g({{i}_{2}})/{{V}_{s}}}{\mu } \right)=\frac{\frac{{{E}_{1}}}{{{R}_{2}}}+\frac{{{V}_{S}}}{{{R}_{2}}}}{\frac{1}{{{R}_{1}}}+\frac{1}{{{R}_{2}}}}\Rightarrow {{V}_{S}}=-\frac{\frac{{{E}_{1}}}{{{R}_{1}}}}{\frac{1}{{{R}_{2}}}+\left( \frac{1+g({{i}_{2}})/{{V}_{s}}}{\mu } \right).\left( \frac{1}{{{R}_{1}}}+\frac{1}{{{R}_{2}}} \right)}=\frac{{{V}_{0}}}{1+K\left( 1+g({{i}_{2}})/V \right)}\] A condition de poser: ${V_0} = - \frac{{{R_2}}}{{{R_1}}}{E_1}{\rm{ et K = }}\frac{{{{\rm{R}}_{\rm{2}}}}}{\mu }(\frac{1}{{{R_1}}} + \frac{1}{{{R_2}}})$
Si on fait µ → ∞ on retrouve bien le résultat de la question II-2) puisqu'alors K = 0
Si on fait g(i2) ≡ 0 alors on retrouve le résultat de la question II-5), ce qui est satisfaisant.
II-7)Le coefficient K est très faible car le coefficient µ est très grand pour un ampli-op, cela veut dire que l'influence des arrondis sur le comportement réel vis à vis du comportement idéal des diodes est négligeable avec un montage à ampli-op.

III Fonctions à seuil à plusieurs cassures.
III-1) Le théorème de Millman donne immédiatement:
${V^ - } = {V^ + } = 0 = \frac{{\frac{{{V_{S1}}}}{{{R_1}}} + \frac{{{V_{S2}}}}{{{R_2}}} + \frac{{{V_S}}}{{{R_3}}}}}{{\frac{1}{{{R_1}}} + \frac{1}{{{R_2}}} + \frac{1}{{{R_3}}}}}$ soit ${V_S} = - {R_3}(\frac{{{V_{S1}}}}{{{R_1}}} + \frac{{{V_{{S_2}}}}}{{{R_2}}})$
III-2) Les diodes D1 et D2 sont montées têtes bêches deux cas sont à examiner:
Ier Cas: D1 est passante et D2 est bloquée. ⇔ VS1 = V- = 0 (D1 passante)
Alors en vertu du théorème de Millman si D2 est bloquée:
$\frac{{{V_1}}}{R} + \frac{E}{R} = {I_{D1}} > 0$ ⇒ E > -V1
2ème Cas: D1 est bloquée et D2 est passante. ⇔ conséquence sur le théorème de Millman
${V^ - } = 0{\rm{ }} \Rightarrow {\rm{ 0}} = \frac{{{V_1}}}{R} + \frac{{{V_{S1}}}}{R} + \frac{E}{R}$ ⇒ VS1 =-(E+V1) or VS1 < V- = 0 soit E < -V1
Les deux cas possibles de valeurs de E sont donc passés en revue.
III-3)
Cette fois les diodes D3 et D4 sont montées différemment.
On a: V- = V+ = 0
U3 +U4 =VS2 -V- = VS2
Aux bornes d'une diode idéale la tension ne peut être que nulle ou négative, il s'en suit que VS2 ne peut être que négatif ou nul.
Alors les courants I et I0 de la figure seront positifs ou nuls tous les deux
Ier Cas: D3 et D4 sont passantes. ⇔ U3 = U4 = 0 ⇒ VS2 = 0 et I = I0 = 0
${i_3} = - \frac{E}{R} + \frac{{{V_2}}}{R}{\rm{ > 0}}$ (D3 passante) donc il faut que E < V2
2eme Cas: D3 est bloquée - D4 est passante. ⇔ i3 = 0
et conséquence du théorème de Millman: ${{\rm{V}}^{\rm{ - }}} = 0{\rm{ }} \Rightarrow {\rm{ 0}} = \frac{{ - {V_2}}}{R} + \frac{{{V_{S2}}}}{R} + \frac{E}{R}$
VS2 = V2 -E la condition VS2 < 0 impose d'avoir E > V2
3eme Cas: D3 est passante - D4 est bloquée ⇔ i4 = I + I0 = 0 ⇒ I = I0 = 0 et VS2 est nul
${i_3} = - \frac{E}{R} + \frac{{{V_2}}}{R}{\rm{ > 0}}$ (D3 passante) donc il faut que E < V2
4eme Cas: D3 et D4 sont bloquées. ⇔ i4 = I + I0 = 0 ⇒ I = I0 = 0 et VS2 est nul
i3 = 0 (D3 bloquée) il faut que E = V2
Il y a donc deux situations: $\left\{ \begin{array}{l}E > {V_2}{\rm{ }} \Rightarrow {\rm{ }}{{\rm{V}}_{{\rm{S2}}}} = {V_2} - E\\E \le {V_2}{\rm{ }} \Rightarrow {\rm{ }}{{\rm{V}}_{{\rm{S2}}}} = 0\end{array} \right.$
III-4)On fait la synthèse des études précédentes sous forme d'un tableau:
E E < -V1 V1 < E < V2 E > V2
VS1 -(V1 + E) 0 0
VS2 0 0 (V2 - E)
${V_S} = - {R_3}(\frac{{{V_{S1}}}}{{{R_1}}} + \frac{{{V_{S2}}}}{{{R_2}}})$ $\frac{{{R_3}}}{{{R_1}}}({V_1} + E)$ 0 $ - \frac{{{R_3}}}{{{R_2}}}({V_2} - E)$
Ce qui se représente graphiquement par la caractéristique VS =f(E) ci-contre.
La caractéristique obtenue est celle d'une pseudo-diode Zener (cf figure 1) avec la correspondance des grandeurs:
(U,I) → (E,VS)
L'intérêt de ce montage est de réduire les défauts dûs aux arrondis.
III-5) La tension de sortie VS est indépendante du courant de sortie, on a donc en sortie une source de tension parfaite d'impédance de sortie ZS = 0
• • • • • • • • • • • • • •
DEUXIÈME PROBLÈME - MÉCANIQUE
I Étude préliminaire
I-1)
Dans un triangle équilatéral une rotation d'angle 2π/3 autour du point intersection des médianes (hauteur, médiatrices, etc..) laisse invariant le triangle et lui même. Il s'agit donc du centre d'inertie de la plaque homogène.
Sa position est: $AG = \frac{{2h}}{3}$
I-2)
Le moment d'inertie est ici un calcul d'intégrale double:
$I(Ax) = \int_0^h {dy\int_{ - y\tan (30^\circ )}^{y\tan (30^\circ )} {{y^2}.dx} } $
(en convenant que la masse surfacique est unitaire M≡S)
$I(Ax) = 2\int_0^h {dy\left[ {x{y^2}} \right]_0^{y\tan (30^\circ )}} = \frac{2}{{\sqrt 3 }}\left[ {\frac{{{y^4}}}{4}} \right]_0^h$
$I(Ax) = \frac{2}{{\sqrt 3 }}\left[ {\frac{{{h^4}}}{4}} \right]{\rm{ or M}} \equiv {\rm{S = }}\frac{{{{\rm{h}}^{\rm{2}}}}}{{\sqrt 3 }}$ donc $I(Ax) = \frac{{M{h^2}}}{2}$
A-N: I(Ax) = 2,5.10-3 kg.m2
I-3)Les trois côtés sont équivalents pour ce calcul. Donc J(AB) = J(BC). On peut utiliser deux fois le théorème de Huyghens pour calculer ce dernier.
$J(BC) = \left( {I(Ax) - M{{\left( {\frac{{2h}}{3}} \right)}^2}} \right) + M{(\frac{h}{3})^2}$
$J(AB) = \frac{{M{h^2}}}{6}$ A-N: J(AB) = 8,4.10-4 kg.m2
I-4)
Nous calculerons d'abord le moment d'inertie I(Az) puis par le théorème de Huyghens on obtiendra K(Gz).
$I(Az) = \int_0^h {dy\int_{ - y\tan (30^\circ )}^{y\tan (30^\circ )} {({x^2} + {y^2}).dx} } = 2\int_0^h {dy\left[ {\frac{{{x^3}}}{3} + x{y^2}} \right]} _0^{y\tan (30^\circ )}$ $I(Az) = 2\int_0^h {dy\left[ {\frac{{{y^3}}}{{9\sqrt 3 }} + \frac{{{y^3}}}{{\sqrt 3 }}} \right]} = \frac{{20}}{{9\sqrt 3 }}\left[ {\frac{{{y^4}}}{4}} \right]_0^h$
$Avec{\rm{ M}} \equiv {\rm{S = }}\frac{{{{\rm{h}}^{\rm{2}}}}}{{\sqrt 3 }}{\rm{ }} \Rightarrow {\rm{ }}I(Az) = \frac{{20M{h^2}}}{{36}}$ donc
${\rm{K}}(Gz) = I(Az) - M{(\frac{{2h}}{3})^2} = \frac{{20M{h^2}}}{{36}} - \frac{{4M{h^2}}}{9}$ ⇒ ${\rm{K}}(Gz) = \frac{{M{h^2}}}{9}$ A-N: K(Gz)= 5,6.10-4 kg.m2

II Rotation autour d'un côté du triangle
On a affaire à un pendule pesant se déplaçant sans frottement. Il y a conservation de l'énergie mécanique de la plaque. Compte tenu du sens d'orientation de l'axe vertical l'énergie s'écrit:
${E_m} = \frac{1}{2}{J_{AB}}{\dot \alpha ^2} - Mg{z_G} = \frac{{M{h^2}}}{{12}}{\dot \alpha ^2} - Mg\frac{h}{3}\cos \alpha = {C^{te}}$
D'où le résultat cherché ${\dot \alpha ^2} = \frac{{4g}}{h}\cos \alpha $
Le mouvement de G est circulaire, il y donc une accélération centripète et une accélération tangentielle. Au passage par la verticale l'accélération tangentielle est nulle puisque la vitesse est maximale. Il reste alors uniquement la composante centripète que l'on calcule pour α=0
${a_G} = \frac{h}{3}{\dot \alpha ^2} = \frac{{4g}}{3}$ A-N: aG = 13,1 m.s-2
III Mouvement de vissage
III-1)
L'angle θ de rotation du solide est représenté dans le plan de la plaque par l'angle entre GA' et GA. A' est l'intersection de la verticale A0z avec le plan de la plaque. On peut écrire:
$AA' = 2.\frac{{2h}}{3}\sin (\theta /2)$
Cette seconde figure représente la situation dans un plan vertical des points A, A0 et fait apparaître l'ascension du point A jusqu'en A' tel que A0A' = L - zG
La longueur constante du fil se traduit par:
${L^2} = {({z_G})^2} + {(AA')^2}$
${L^2} = z_G^2 + {[\frac{{4h}}{3}\sin (\theta /2)]^2}$
III-2)
D'après le théorème de König ${E_c} = \frac{1}{2}M\dot z_G^2 + \frac{1}{2}{K_{Gz}}{\dot \theta ^2} = \frac{1}{2}M\dot z_G^2 + \frac{{M{h^2}}}{{18}}{\dot \theta ^2}$

III-3)Dans l'hypothèse des petits mouvements on a θ voisin de 0 et zG voisin de L. La relation géométrique devient
${L^2} = z_G^2 + {[\frac{{4h}}{3}\sin (\theta /2)]^2}$ ⇒ ${L^2} = {(L - \varepsilon )^2} + \frac{{4{h^2}{\theta ^2}}}{9}$
Et en dérivant on obtient: $0 = - 2\dot \varepsilon (L - \varepsilon ) + \frac{{8{h^2}\dot \theta \theta }}{9} \approx - 2\dot \varepsilon L + \frac{{8{h^2}\dot \theta \theta }}{9}$
Tandis que la relation de définition de l'énergie cinétique devient:
${E_c} = \frac{1}{2}M{\dot \varepsilon ^2} + \frac{{M{h^2}}}{{18}}{\dot \theta ^2}$ soit en éliminant ε, ${E_c} = \frac{1}{2}M\left[ {\frac{{16{h^4}{{\dot \theta }^2}{\theta ^2}}}{{81{L^2}}} + \frac{{{h^2}{{\dot \theta }^2}}}{9}} \right] = \frac{{M{h^2}}}{{18}}{\dot \theta ^2}\left[ {1 + \frac{{16{h^2}{\theta ^2}}}{{9{L^2}}}} \right]$
Le terme en θ2 est négligeable et on a alors pratiquement ${E_c} = \frac{{M{h^2}}}{{18}}{\dot \theta ^2}$
L'énergie potentielle est Ep =-MgzG =-Mg(L-ε), on écrit alors la constance de l'énergie mécanique:
${E_m} = - Mg(L - \varepsilon ) + \frac{{M{h^2}}}{{18}}{\dot \theta ^2} = {C^{te}}$ par dérivation ⇒ $\frac{{d{E_m}}}{{dt}} = Mg\dot \varepsilon + \frac{{M{h^2}}}{{18}}2\dot \theta \ddot \theta = 0$
Il reste encore à remplacer ε
$Mg\frac{{4{h^2}\dot \theta \theta }}{{9L}} + \frac{{M{h^2}}}{{18}}2\dot \theta \ddot \theta = 0{\rm{ }}$ l'équation des petits mouvements est ${\rm{ }}\ddot \theta {\rm{ + }}\frac{{{\rm{4g}}}}{{\rm{L}}}\theta = 0$
Il s'agit de mouvements sinusoïdaux de période: $T = 2\pi \sqrt {\frac{L}{{4g}}} $ A-N: T= 316 ms
• • • • • • • • • • • • • •

Concours Physique EIVP P' 1994 (Corrigé)

I.V.P. 1994 option P' Freinage d'une navette par l'atmosphère
Interaction entre deux spires.
1 Préliminaire
$p = \mu \frac{{RT}}{M}$ atmosphère en équilibre isotherme $ \Rightarrow \mu * \vec g = gra\vec d\left( p \right)$
$\frac{\partial p}{\partial z}=-\text{ }\mu *g\text{ }\xrightarrow{{}}\text{ }\frac{\partial \mu }{\partial z}=-\text{ }\mu \cdot g\frac{M}{RT}$ $\mu = {\mu _S} \cdot \exp \left( { - {\rm{ }}\frac{{Mgz}}{{RT}}} \right) = {\mu _S}\exp \left( { - {\rm{ }}\frac{z}{d}} \right)$
$d = \frac{{RT}}{{Mg}} = {\rm{ }}8000m{\rm{ }} \Rightarrow {\rm{ }}$$T = 279{\rm{ }}K$
2 Freinage vertical $\frac{dv}{d\mu }\text{ + }\frac{{{\text{C}}_{\text{1}}}\cdot d}{m}\cdot v\text{ = 0}$
2.1. $\frac{{dv}}{{dt}} = {\rm{ - }}\mu \cdot \frac{{{{\rm{C}}_{\rm{1}}}}}{{\rm{m}}} \cdot v{{\rm{ }}^2}{\rm{ = }}\frac{{{\rm{dv}}}}{{{\rm{d}}\mu }} \cdot \frac{{d\mu }}{{dh}} \cdot \frac{{dh}}{{dt}}$
avec $\frac{{dh}}{{dt}}{\rm{ }} = {\rm{ }} - v{\rm{ ; et }}\frac{{{\rm{d}}\mu }}{{{\rm{dh}}}}{\rm{ = - }}\frac{\mu }{{\rm{h}}}$
2.2 dv/v = -(C1 .d/m).dµ Ln(v/v0) = C1 .d/m.(µ0-µ) $v{\rm{ = }}{{\rm{v}}_{\rm{0}}} \cdot \exp \left( {\frac{{d.{C_1}}}{m} \cdot ({\mu _0} - \mu )} \right)$

2.3 A l'altitude h0 la masse volumique est très faible (µ0 = 4,8.10-6) et le freinage très peu efficace; dans le cadre de ce modèle très grossier la vitesse à l'arrivée au sol est très faible: v= v0* exp(-20,8) = 7,4 .10-6 m/s
bien, sur la force de freinage, qui varie comme le carré cette vitesse, n'est plus efficace; qualitativement on voit que l'efficacité du freinage passe par un maximum; il resterait à définir quantitativement cette " efficacité".
2.4 δ = -dv/dt = µ(h).C1 /m.V2 = .C1 /m.V02 µ.exp{ 2.C1 (d./m).(µ0-µ)}
il y aurait un maximum de décélération là où d δ /dµ. = 0 = [1 - µ. 2.C1 d./m.]* δ /µ
soit quand ${\mu _M}{\rm{ = }}\frac{{\rm{m}}}{{{\rm{2}} \cdot {\rm{d}} \cdot {{\rm{C}}_{\rm{1}}}}}{\rm{ }} = {\rm{ }}\frac{{{\rm{5}}{\rm{.1}}{{\rm{0}}^{\rm{3}}}}}{{{{8.10}^3}.2.10}}{\rm{ = }}\frac{{\rm{1}}}{{{\rm{32}}}}kg/{m^3}$
ce qui correspond à une altitude ${h_M}{\rm{ = d}} \cdot {\rm{Ln(}}\frac{{{\mu _{\rm{S}}}}}{{{\mu _{\rm{M}}}}}){\rm{ = 8000}}{\rm{.Ln42 = 29}}{\rm{,9km}}$
la décélération serait $\delta _M^{}{\rm{ = }}\mu _M^{} \cdot \frac{{{C_1}}}{m} \cdot V_0^2 \cdot \frac{1}{e}{\rm{ = }}\frac{{\rm{1}}}{{\rm{e}}} \cdot \frac{{V_0^2}}{{2.d}}{\rm{ = 1470 m}} \cdot {{\rm{s}}^{{\rm{ - 2}}}}$
soit, dans le cadre de ce modèle, environ 150 fois G.
2.5 Si l'on réintroduit l'attraction terrestre son effet est notable au départ car µ est faible, la vitesse va donc augmenter; on peut majorer cette augmentation en évaluant la vitesse de la navette au sol s'il n'y avait pas d'atmosphère(v '= (64.106+10.105)½ =8,06 km/s
Sous l'action de la gravitation la navette atteindrait une vitesse limite ${V_l}{\rm{ = }}\sqrt {\frac{{{\rm{m}}{\rm{.G}}}}{{\mu .{C_1}}}} $ qui correspond en h=0, où µ = 1,3kg/m3, à Vl = 62 m/s. Dans le cadre du modèle où l' on néglige la gravitation cette vitesse de 62m/s correspond à une altitude calculable par les relations du § 2.2 : µ(h) =m /.(C1 d.).Ln(8000/62) = 0,30 kg/m3, on aurait cette masse volumique donc cette vitesse(62 m/s) en h= d.Ln(1,3/0,3) soit en h = 11,6 km ,(13,2km en tenant compte de µ(h)), altitude nettement inférieure à celle (environ 30 km) où l' on prévoyait une décélération maximale de 150* G, qu'il ne faut pas faire subir à d'hypothétiques passagers. L'allure générale de la courbe v=f(h), et ses conséquences, sont donc peu modifiées.

3 Freinage sur une spirale
3.1 On projette sur la tangente à la trajectoire la relation fondamentale en "oubliant" là encore le terme de gravitation lié à la terre(m.G.cosα) : $\frac{{dv}}{{dt}} = {\rm{ - }}\mu \cdot \frac{{{{\rm{C}}_{\rm{1}}}}}{{\rm{m}}} \cdot v{{\rm{ }}^2}{\rm{ = }}\frac{{{\rm{dv}}}}{{{\rm{d}}\mu }} \cdot \frac{{d\mu }}{{dh}} \cdot \frac{{dh}}{{dt}}$
relation inchangée, mais avec dh /dt = - Vcosα $\frac{{dv}}{{d\mu }}{\rm{ + }}\frac{{{{\rm{C}}_{\rm{1}}} \cdot d}}{{m.\cos \alpha }} \cdot v{\rm{ = 0}}$
3.2 Le freinage sur l'air raréfiée de la très haute atmosphère provoque une perte lente d'énergie mécanique pour un satellite, même en orbite circulaire (α=π / 2) et donc une lente diminution de l'altitude. Ce phénomène n'est pas pris en compte ici. Donc si α=π / 2 il n'y a pratiquement pas freinage et si α=0 la rentrée est la plus "brutale".
3.3 On observera le maximum de décélération pour d δ /dµ.=[1 - µ. 2.C1 d./m.cosα.].( δ /µ)
La décélération maximale sera $\Delta _{M}^{'}=\mu _{M}^{'}.\frac{{{C}_{t}}}{m}.V_{0}^{2}.\frac{1}{e}=\frac{\cos \alpha }{e}.\frac{V_{0}^{2}}{2.d}=1470.\cos \alpha $ pour qu'elle soit inférieure à 10.G il faudra cosα < 1 / 14,7 soit π / 2 > α > π / 2 - 0,068
La longueur L de la trajectoire parcourue par la navette sera : $L{\rm{ = }}\int_{{\rm{t = 0}}}^{{{\rm{t}}_{\rm{F}}}} {{\rm{v}}{\rm{.dt}}} {\rm{ = }}\int_{{\rm{t = 0}}}^{{{\rm{t}}_{\rm{F}}}} {\frac{{{\rm{dh}}}}{{{\rm{dt}}}} \cdot \frac{{\rm{1}}}{{{\rm{cos}}\alpha }}{\rm{dt}}} {\rm{ = 14}}{\rm{,7}}{\rm{.}}{{\rm{h}}_{\rm{0}}}{\rm{ = 1470 km}}$
3.4 On néglige, lors de la descente de la navette, la perte d'énergie potentielle gravitationnelle devant la perte d'énergie cinétique 64 fois plus importante:-ΔEc =1/2.m.v²=1,6.1011 joules
Pour dissiper cette énergie on songe à la vaporisation d'une céramique; il en faudrait:
$ - \Delta {E_c}{\rm{ = }}\left( {{{\rm{I}}_{{\rm{Fus}}}} + {I_{vap}}} \right) \cdot {{\rm{m}}_{{\rm{ceram}}}}{\rm{ Soit }}{{\rm{m}}_{{\rm{ceram}}}}{\rm{ = }}\frac{{{\rm{1}}{\rm{,6}}{\rm{.1}}{{\rm{0}}^{{\rm{11}}}}}}{{{{10}^7}}}{\rm{ = 16 tonnes }}$
Cette valeur est bien sur incohérente avec la valeur, 5 tonnes, de la masse de la navette; on peut penser qu'il y a en plus évacuation de la chaleur par convection et surtout par rayonnement, l'importance de ces facteurs augmente.si la durée du vol spirale croît(α→π / 2)
Pb 2 : INTERACTION ENTRE DEUX SPIRES
1 Etude des phénomènes électromagnétiques
1.1 ${B_z}{\rm{ = }}\frac{{{\mu _{\rm{0}}}{I_1}}}{{2a}}{\left( {1 + \frac{{{a^2}}}{{{z^2}}}} \right)^{ - \frac{3}{2}}} \cong {\rm{ }}\frac{{{\mu _{\rm{0}}}{I_1}{a^2}}}{{2{z^3}}}$
1.2 L'Inductance mutuelle M entre les deux spires:
$M{\rm{ = }}\frac{{{\Phi _{{\rm{12}}}}}}{{{{\rm{I}}_{\rm{1}}}}}{\rm{ }} \cong {\rm{ }}\frac{{{\mu _{\rm{0}}}\pi {a^4}}}{{2{z^3}}}{\rm{ }}$ et${I_{2{\rm{ }}}}{\rm{ = - }}\frac{{\rm{1}}}{{\rm{R}}}\frac{{d{\Phi _{12}}}}{{dt}}{\rm{ = }}\frac{{\rm{1}}}{{\rm{R}}}\frac{{3{\mu _0}\pi {a^4}}}{{2{z^4}}}{I_1}\left( {\frac{{dz}}{{dt}}} \right)$
M≠0 alors que L1 et L2 sont nulles peut surprendre. On peut espérer qu'un candidat s'en étonnant et invoquant L2.L1 ≥ M2 aura été fortement récompensé !
1.3 . a
1°) Symétrie de révolution autour de oz ⇒ $\vec B{\rm{ }}$ indépendant de θ
2°) Le plan $M,{\vec u_r},{\vec u_\theta }$ est plan de "symétrie négative" ⇒ $\vec B{\rm{ (M) }} \in {\rm{ au plan }}M,{\vec u_r},{\vec u_\theta }$
1.3 . b $2\pi .r.dz.{B_r} + \pi .{r^2}.\frac{{\partial {B_z}}}{{\partial z}}.dz{\rm{ = 0}}$${B_r}{\rm{ = - }}\frac{{\rm{a}}}{{\rm{2}}}\frac{{\partial {B_z}}}{{\partial z}}{\rm{ = }}\frac{{\rm{a}}}{{\rm{2}}} \cdot \frac{{3{\mu _0}\pi {a^2}{I_1}}}{{2{z^4}}}$
1 3 . c $\vec F{\rm{ = }}\oint {{{\rm{I}}_{\rm{2}}}.d\vec l \wedge \vec B} {\rm{ = - }}{{\rm{\vec u}}_{\rm{z}}}.2\pi a.{B_r}$
${F_z}{\rm{ = - }}\left( {\frac{{\rm{a}}}{{\rm{2}}} \cdot \frac{{3{\mu _0}\pi {a^2}{I_1}}}{{2{z^4}}}} \right)\frac{{\left( {2\pi a} \right)}}{{\rm{R}}}\frac{{3{\mu _0}\pi {a^4}{I_1}}}{{2{z^4}}}\left( {\frac{{dz}}{{dt}}} \right){\rm{ = - }}\frac{{\rm{1}}}{{{\rm{R}}{\rm{.}}{{\rm{z}}^{\rm{8}}}}}{\left( {\frac{{3{\mu _0}\pi {a^4}}}{2}} \right)^2}\frac{{dz}}{{dt}}$
prend la forme demandée avec $k{\rm{ = }}\frac{{\rm{1}}}{{{\rm{2}}{\rm{.m}}{\rm{.R}}}} \cdot {\left( {3{\mu _0}\pi {a^4}I} \right)^2}$ Bien sur $\vec F{{\rm{ }}_{{\rm{12}}}}{\rm{ = - }}\vec F{{\rm{ }}_{{\rm{21}}}}$

2 Etude des mouvements des spires
2.1 $\begin{array}{l}\\\frac{{{d^2}{z_1}}}{{d{t^2}}} + \frac{{{d^2}{z_2}}}{{d{t^2}}}{\rm{ = 0 ; }}\frac{{{{\rm{d}}^{\rm{2}}}{z_2}}}{{d{t^2}}}{\rm{ = - }}\frac{{\rm{k}}}{{{\rm{2}}{\rm{.}}{{\rm{z}}^{\rm{8}}}}} \cdot \frac{{dz}}{{dt}}{\rm{ = - }}\frac{{{{\rm{d}}^{\rm{2}}}{z_1}}}{{d{t^2}}}{\rm{ = }}\mathop {\frac{1}{2}}\limits^{..} \frac{{{d^2}z}}{{d{t^2}}}\end{array}$
et $\frac{{dz}}{{dt}}{\rm{ = }}\frac{{\rm{k}}}{{{\rm{7}}{\rm{.}}{{\rm{z}}^{\rm{7}}}}}{\rm{ + C = }}\frac{{\rm{k}}}{{{\rm{7}}{\rm{.z}}_{\rm{0}}^{\rm{7}}}}\left( {\frac{{{\rm{z}}_{\rm{0}}^{\rm{7}}}}{{{{\rm{z}}^{\rm{7}}}}}{\rm{ - }}1} \right){\rm{ + }}{{\rm{v}}_{\rm{0}}}{\rm{ = g(z) }}$
2.2. a d²z /dt² est négatif à la date t = 0, puisque v0 > 0; mais selon la valeur de v0 et surtout du signe de (7.v0.z07-k ), deux cas sont possibles; dans le premier g(z→∝)>0
alors(7.v0.z07-k)>o,
pour g(z→∝)>0 :
Le régime permanent lorsque t→ ∝ est un mouvement uniforme à la vitesse ${v_\infty }{\rm{ = }}{{\rm{v}}_{\rm{0}}}{\rm{ }} - {\rm{ }}\frac{k}{{7.z_0^7}}$ ; les spires s'écartent indéfiniment.
2.2 . b Si maintenant g(z→∝) < 0, c'est à dire qu'alors (7.v0.z07-k) < 0, la vitesse d'éloignement, s'annule pour une valeur finie de z; c'est le point F du graphe inférieur. Les spires s'immobilisent et comme on a toujours $\mathop {\rm{z}}\limits^{ \bullet {\rm{ }} \bullet } \cdot \mathop {\rm{z}}\limits^ \bullet {\rm{ }} \le {\rm{ 0}}$, le mouvement, ne peut reprendre.
2.2 . c La courbe intermédiaire du graphe est bien la courbe séparatrice du diagramme des phases.
2.3 D'un point de vue énergétique on peut écrire que la variation de la somme de l'énergie magnétostatique et de l'énergie cinétique deux spires est égale à la somme des énergies reçues de l'extérieur c'est à dire ici au travail du générateur qui maintient le courant ${I_1}$ constant et la chaleur " reçue"de l'extérieur , algébriquement négative, contrepartie de l'effet joule. On peut aussi écrire le théorème de l'énergie cinétique: c'est à dire: variation de l'énergie cinétique des deux spires égale au travail de toutes les forces, ici les forces de Laplace sur les deux spires. Il reste une difficulté relative à l'état initial de la seconde bobine- (à t = o, il est possible de considérer ${{\rm{I}}_{\rm{2}}}$ = 0, il faudra alors un L faible mais non nul, ou ${{\rm{I}}_{\rm{2}}}$ ≠ 0, cela à quelques répercutions sur le bilan;

A noter que $\int {d(M.{I_1}.{I_2}} ){\rm{ = }}{I_1}.\Delta (M.{I_2}) = - M.{I_1}.{I_{{2_0}}} = - {\rm{ }}\frac{{7m.v_0^2}}{{12}}$, n'est pas négligeable)
si 7.v0.z07 = 2k les spires s'éloignent indéfiniment (§ 2.2.a) et dz/dt= (v0/2).(1+ z07 /z7)
Travail des forces de Laplace:
${W_L}{\rm{ = - }}\int\limits_{{\rm{z = }}{{\rm{z}}_{\rm{0}}}}^\infty {\frac{{{\rm{k}}{\rm{.m}}}}{{\rm{2}}} \cdot \frac{1}{{{z^8}}}} \cdot \frac{{{\rm{dz}}}}{{{\rm{dt}}}} \cdot {\rm{dz = - (m}}{\rm{.}}{{\rm{v}}_{\rm{0}}} \cdot \frac{{\rm{7}}}{{\rm{4}}}{\rm{)}} \cdot \frac{{{{\rm{v}}_{\rm{0}}}}}{{\rm{2}}}\int\limits_{{\rm{u = 1}}}^\infty {\left( {{{\rm{u}}^{{\rm{ - 15}}}}{\rm{ + }}{{\rm{u}}^{ - 8}}} \right)} {\rm{ du = }}\underline {{\rm{ - }}\frac{{\rm{3}}}{{{\rm{16}}}} \cdot {\rm{mv}}_{\rm{0}}^2} {\rm{ }}$
énergie joule:
${W_J}{\rm{ = }}\int\limits_{t = 0}^\infty {{\rm{R}}{\rm{.}}{{\rm{I}}_{\rm{2}}}{{(t)}^2}} {\rm{.dt = }}\int {{\rm{R}}{\rm{.}}} {\left( {\frac{{\rm{1}}}{{\rm{R}}}\frac{{3{\mu _0}\pi {a^4}}}{{2{z^4}}}{I_1}\left( {\frac{{dz}}{{dt}}} \right)} \right)^{\rm{2}}}{\rm{dt = }}\int {\frac{{{\rm{k}}{\rm{.m}}}}{{{\rm{2}}{\rm{.}}{{\rm{z}}^{\rm{8}}}}}{{\left( {\frac{{dz}}{{dt}}} \right)}^2} \cdot } {\rm{ dt = - }}{{\rm{W}}_{\rm{L}}}$
variation d'énergie cinétique:
${E_{{c_{initiale}}}}{\rm{ = 2}}{\rm{.}}\left( {\frac{{\rm{m}}}{{\rm{2}}} \cdot {{(\frac{{v_0^{}}}{2})}^2}} \right){\rm{ = }}\frac{{{\rm{m}}{\rm{.v}}_{\rm{0}}^{\rm{2}}}}{{\rm{4}}}{\rm{ ; }}{{\rm{E}}_{{c_{finale}}}}{\rm{ = 2}}{\rm{.}}\left( {\frac{{\rm{m}}}{{\rm{2}}} \cdot {{(\frac{{v_0^{}}}{4})}^2}} \right){\rm{ = }}\frac{{{\rm{m}}{\rm{.v}}_{\rm{0}}^{\rm{2}}}}{{{\rm{16}}}}{\rm{ }} \Rightarrow {\rm{ }}\underline {\Delta {{\rm{E}}_{{\rm{c }}}}{\rm{ = }}\frac{{{\rm{3}}{\rm{.m}}{\rm{.v}}_{\rm{0}}^{\rm{2}}}}{{{\rm{16}}}}} $

Concours Physique Centrale-Supélec (M, P') 1993 (Énoncé)

Centrale–Supélec, M, P’, 1993 (Physique I)
Énoncé
Ce problème comporte trois parties dont certaines questions peuvent être abordées de façon indépendante. La première partie abordera la propagation d’une onde de courant dans une ligne électrique, la deuxième précisera la structure du champ électromagnétique dans la ligne et la troisième traitera de la transmission d’une onde électromagnétique par une lame conductrice. Les données numériques sont regroupées en fin d’énoncé; on posera \(j^2 = -1\).

Onde de courant dans une ligne électrique

Une ligne électrique sans pertes est caractérisée par son coefficient d’inductance propre linéique et sa capacité linéique, respectivement notées \(L\) et \(C\). À l’abscisse \(x\) et à l’instant \(t\), on désigne par \(i(x,t)\) l’intensité du courant dans la ligne et par \(u(x,t)\) la tension entre les deux conducteurs de la ligne (cf. fig. [fig1]).
  1. Établir les deux équations différentielles liant \(i(x,t)\) et \(u(x,t)\).
  2. [I2] On cherche une solution de ces équations représentant une onde de courant de la forme \(i(x,t) = I(x) \exp \left(j \omega t\right)\) en notation complexe. Déterminer, dans ce cas, la forme la plus générale de \(i(x,t)\) et \(u(x,t)\). Exprimer en fonction des caractéristiques de la ligne la vitesse de phase \(v_\varphi\) de cette onde.
  3. La ligne, située dans l’espace \(x < 0\), s’étend jusqu’en \(x = 0\) où elle est fermée sur l’impédance \(Z_0\) (cf. fig. [fig2]). Montrer qu’il existe une valeur \(Z_c\) de \(Z_0\), appelée impédance caractéristique de la ligne telle que le rapport \(u/i\) devienne indépendant de \(x\). On exprimera \(Z_c\) en fonction de \(L\) et \(C\) et on précisera la forme de l’onde dans la ligne. Exprimer dans ce cas la puissance moyenne transportée par l’onde à l’abscisse \(x\). Que se passe-t-il physiquement en \(x = 0\)?
  4. La ligne s’étend maintenant jusqu’à \(x = + \infty\) mais on branche encore l’impédance \(Z_0 = Z_c\) en parallèle sur la ligne à l’abscisse \(x = 0\) (cf. fig. [fig3]). On s’intéresse à l’onde de courant dans la partie \(x < 0\) de la ligne.
    1. Montrer que cette onde voit en \(x = 0\) une impédance équivalente \(Z_1\) qui s’exprime très simplement en fonction de \(Z_c\).
    2. Définir et calculer le module \(r\) du coefficient de réflexion (en courant ou en tension) de l’onde en \(x = 0\).
  5. On place enfin sur la ligne précédente un court-circuit en parallèle à l’abscisse \(x = \ell\) (cf. fig. [fig4]).
    1. Quelle est la forme nécessaire de l’onde de courant entre les abscisses \(x=0\) et \(x=\ell\)?
    2. Montrer qu’il existe une valeur minimale \(\ell_0\) de \(\ell\) telle que le courant dans la partie positive de la ligne s’annule en \(x=0\). On exprimera \(\ell_0\) en fonction de la longueur d’onde \(\lambda\) de l’onde de courant dans la ligne. En déduire alors le coefficient de réflexion et la forme de l’onde dans la partie négative de la ligne.

Champ électromagnétique dans la ligne

La ligne précédente est constituée de deux rubans conducteurs parfaits, de faible épaisseur, de largeur \(a\), distants de \(b\), l’espace entre les rubans étant vide (cf. fig. [fig5]). Les rubans sont parcourus par des courants de densités surfaciques \(\vec j_s = j_s(x,t) \vec e_x\) et \(- \vec j_s\) et présentent entre leurs faces des densités surfaciques de charge \(\sigma(x,t)\) et \(- \sigma(x,t)\).
On étudie les champs \(\vec E\) et \(\vec B\) uniquement dans l’espace situé entre les rubans et on suppose que ces champs ne dépendent que l’abscisse \(x\) du point considéré et de l’instant \(t\). On néglige donc tout effet de bord.
  1. Exprimer, en fonction des constantes électromagnétiques du vide \(\varepsilon_0\) et \(\mu_0\) et des densités \(j_s\) et \(\sigma\) les champs \(\vec E(x,t)\) et \(\vec B(x,t)\) dans l’espace vide entre les rubans.
    On considère à nouveau dans toute la suite de cette partie [PartieII] une onde de courant dans la ligne, d’intensité de la forme \(i(x,t) = I \exp \left[j \left(\omega t - k x\right)\right]\) en notation complexe, où \(k\) est une constante positive et \(I\) une constante réelle.
  2. [II2] À partir des équations de Maxwell, exprimer deux relations liant \(\sigma(x,t)\) et \(i(x,t)\). En déduire la vitesse de phase \(v_\varphi\) de l’onde et montrer que la structure du champ électromagnétique est celle d’une onde plane dans le vide illimité.
  3. Déterminer l’énergie magnétique \({\mathrm{d}}\epsilon_B\) d’une tranche d’épaisseur \({\mathrm{d}}x\) de la ligne. En déduire le coefficient d’inductance propre \(L\) de la ligne.
  4. Déterminer l’énergie \({\mathrm{d}}\epsilon_E\) associée au champ électrique \(\vec E\) de la même tranche d’épaisseur \({\mathrm{d}}x\). En déduire la capacité linéique \(C\) de la ligne.
  5. Déduire des résultats précédents l’accord entre les questions [I2] et [II2] du problème quant à la vitesse de phase \(v_\varphi\).
  6. Exprimer le champ \(\vec E\) en fonction des dimensions de la ligne et de la tension \(u(x,t)\) entre les rubans. Peut-on écrire une relation de la forme \(\vec E = - {\overrightarrow{\mathrm{grad}}\,}V\) dans l’espace vide entre les rubans?
    On désire fermer la ligne sur son impédance \(Z_c\) en introduisant, entre les rubans, à l’abscisse \(x = 0\), une plaque conductrice de résistivité \(\varrho\), d’épaisseur \(e\), de largeur \(a\) et de longueur \(b\) (cf. fig. [fig6]).
  7. On considérera dans cette question que l’épaisseur \(e\) est suffisamment faible pour que l’on puisse admettre que le courant traversant la plaque soit réparti de manière uniforme.
    1. Déterminer \(Z_c\) en fonction de \(\varrho\), \(e\), \(a\) et \(b\). Montrer que la résistance \(R_c\) d’un carré de la plaque, de côté quelconque, s’exprime en fonction des seules constantes \(\varepsilon_0\) et \(\mu_0\). On appellera impédance adaptée au vide cette grandeur \(R_c\) dont on donnera la valeur numérique.
    2. On veut réaliser cette plaque avec:
      • du cuivre de résistivité \(\varrho = {1,7\cdot 10^{-8}}{\,\Omega\cdot\mathrm{m}}\);
      • du carbone de résistivité \(\varrho = {3,5\cdot 10^{-3}}{\,\Omega\cdot\mathrm{m}}\).
      Quel devrait être, dans chaque cas, l’épaisseur \(e\) de la plaque? Commenter.
  8. Déterminer le vecteur de Poynting associé à l’onde électromagnétique entre les rubans. Quelle est la puissance moyenne transportée par l’onde? Que se passe-t-il quand l’onde arrive en \(x = 0\), la ligne étant fermée par la plaque d’impédance \(Z_c\)?

Réflexion sur une plaque conductrice

On considère à présent une onde électromagnétique plane dans le vide illimité, de pulsation \(\omega\) qui a des caractéristiques identiques à celles étudiées dans la partie [PartieII]. On écrira les champs de cette onde:
\[\vec E_i = E_0 \exp \left[j \omega\left(t - \frac{x}{c}\right)\right] \vec e_y \hspace{2em} \vec B_i = \frac{E_0}{c} \exp \left[j \omega\left(t - \frac{x}{c}\right)\right] \vec e_z\]
\(c\) est la vitesse de la lumière dans le vide. À l’abscisse \(x = 0\) (cf. fig. [fig7]) on place une plaque conductrice plane infinie, orthogonale à \(\vec e_x\), de constantes électromagnétiques égales à celles du vide \(\varepsilon_0\) et \(\mu_0\), d’épaisseur \(e\) et de résistivité \(\varrho\) identiques à celles calculées dans la partie précédente: un carré de côté quelconque de la plaque a donc une résistance \(R_c\) adaptée au vide.
  1. Expliquer qualitativement pourquoi il existera pourtant une onde réfléchie sur la plaque. En vous inspirant des résultats précédents et en argumentant votre réponse, pouvez-vous indique sans calculs quel sera le module \(r\) du coefficient de réflexion de cette onde sur la plaque?
    On se propose de retrouver ce résultat directement à partir de l’étude des ondes dans le vide et la plaque. Pour ce faire, on rappelle que, moyennant l’approximation \(\varrho\varepsilon_0\omega \ll 1\) supposée ici vérifiée, le champ électrique dans la plaque conductrice est de la forme:
    \[\vec E_\varrho = \left\{A_1 \exp \left(- \frac{x}{\delta}\right) \exp \left[j \left(\omega t - \frac{x}{\delta}\right)\right] + A_2 \exp \left(\frac{x}{\delta}\right) \exp \left[j \left(\omega t + \frac{x}{\delta}\right)\right] \right\} \vec e_y\]
    \(A_1\) et \(A_2\) sont des constantes déterminées par les conditions aux limites de la plaque et \(\delta\) une distance caractéristique du conducteur et de l’onde, appelée profondeur de peau, et qui vaut \(\displaystyle \delta = \sqrt{\frac{2\varrho}{\mu_0\omega}}\).
  2. Expliquer d’où provient l’approximation indiquée et préciser le champ magnétique \(\vec B_\varrho\) associé dans la plaque. Justifier l’expression de \(\delta\).
    \(\left(\vec E_i, \vec B_i\right)\) étant l’onde incidente arrivant sur la plaque et \(\left(\vec E_\varrho, \vec B_\varrho\right)\) l’onde se propageant dans la plaque, on désigne par \(\left(\vec E_r, \vec B_r\right)\) l’onde réfléchie sur la plaque et \(\left(\vec E_t, \vec B_t\right)\) l’onde transmise dans l’espace \(x > e\).
    On écrira \(\vec E_r\) et \(\vec E_t\) sous la forme:
    \[\vec E_r = \alpha E_0 \exp \left[j \omega\left(t + \frac{x}{c}\right)\right] \vec e_y \hspace{2em} \vec E_t = \tau E_0 \exp \left[j \omega\left(t - \frac{x}{c}\right)\right] \vec e_y\]
  3. Déterminer quatre relations liant \(\alpha\), \(\tau\), \(A_1\) et \(A_2\).
  4. Montrer que l’approximation précédente implique également qu’on ait \(e \ll \delta\). En déduire, après simplifications des relations, la valeur de \(\alpha\).
  5. Que faudrait-il placer, et à quel endroit, pour annuler l’onde réfléchie? On pourra d’abord répondre qualitativement en s’appuyant sur des résultats précédents et démontrer ensuite le résultat recherché.
Formulaire et données numériques:
Formule d’analyse vectorielle \({\overrightarrow{\mathrm{rot}}\,}{\overrightarrow{\mathrm{rot}}\,}\vec u = {\overrightarrow{\mathrm{grad}}\,}{\mathrm{div}\,}\vec u - \Delta \vec u\)
Célérité de la lumière dans le vide \(c = {3,00\cdot 10^{8}}{\,\mathrm{m}\cdot\mathrm{s}^{-1}}\)
Perméabilité magnétique du vide \(\mu_0 = {4\cdot\pi\cdot 10^{-7}}{\,\mathrm{H}\cdot\mathrm{m}^{-1}}\)

Autres Concours

2011  : Concours ENAC de  physique 2011  :  énoncé ,  corrigé Concours ICNA de  physique 2011  :  énoncé ,  corrigé Concours ICNA de ...